LSAT and Law School Admissions Forum

Get expert LSAT preparation and law school admissions advice from PowerScore Test Preparation.

User avatar
 Dave Killoran
PowerScore Staff
  • PowerScore Staff
  • Posts: 5852
  • Joined: Mar 25, 2011
|
#85105
Complete Question Explanation
(The complete setup for this game can be found here: lsat/viewtopic.php?f=351&t=3899)

The correct answer choice is (A)

Answer choice (A) is the correct answer. As explained above, if light 1 is not green, then light 2 cannot be green.

Answer choice (B) is incorrect because, as shown in the correct solution to question #7, when light 1 is yellow, light 2 can be purple.

Answer choice (C) is incorrect because, as shown in the correct solution to question #7, when light 1 is yellow, light 3 can be green.

Answer choice (D) is incorrect because when light 1 is yellow, light 3 can be purple under the following hypothetical: Y-P-P.

Answer choice (E) is incorrect because when light 1 is yellow, light 3 can be yellow under the following hypothetical: Y-P-Y.

Get the most out of your LSAT Prep Plus subscription.

Analyze and track your performance with our Testing and Analytics Package.